Đến nội dung

Idie9xx

Idie9xx

Đăng ký: 30-01-2013
Offline Đăng nhập: 28-10-2021 - 19:34
****-

#544647 $f(x)=k.{log_{10}}^{x}$

Gửi bởi Idie9xx trong 17-02-2015 - 14:01

Cho $f(x)$ là hàm đồng biến trên $(0;+\propto )$ thỏa mãn $f(x_{1}.x_{2})=f(x_{1})+f(x_{2})$ vơi mọi $x$ thuộc $(0;+\propto )$ . chứng minh $f(x)=k.{log_{10}}^{x}$ với $k$ thuộc $N^*$

Đặt $x=e^u$ thì $x_1=e^{u_1},x_2=e^{u_2}$ và $u_1,u_2\in \mathbb{R}$

$\Rightarrow f(e^{u_1+u_2})=f(e^{u_1}.e^{u_2})=f(e^{u_1})+f(e^{u_2})$

Đặt $g(u)=f(e^u)=f(x)$ ta được $g(u_1+u_2)=g(u_1)+g(u_2)$

Do $f$ đồng biến nên $g$ cũng đồng biến kết hợp với cộng tính ở trên $\Rightarrow g(u)=ku$

$\Rightarrow f(x)=ku=k.ln(x)$

Vậy hàm thỏa mãn là $f(x)=k.ln(x),\forall x\in (0,+\propto)$ với $k$ là hằng số thực.




#544552 tìm giá trị $k$ để $f$ là hàm tuần hoàn

Gửi bởi Idie9xx trong 16-02-2015 - 20:56

cho hàm $f:\mathbb{N}\rightarrow \mathbb{R}$ thỏa mãn điều kiện $f(n+1)+f(n-1)=kf(n)$  $\forall n\in \mathbb{N}$

hãy tìm các giá trị của $k$ để $f$ là hàm tuấn hoàn

Spoiler

 

U-Th

Nhìn bài này quen quen hóa ra là giống bài này :) http://diendantoanho...̀m-tuần-hoàn/

Đầu tiên ta đặt $f(n)=u_nf(1)-u_{n-1}f(0), \forall n\geq 2$

Với $u_n$ là dãy thỏa $u_0=0,u_1=1, u_n=ku_{n-1}-u_{n-2}$

Nếu $|k|>2$ thì $u_n=A(\frac{k+\sqrt{k^2-4}}{2})^n+B(\frac{k-\sqrt{k^2-4}}{2})^n$

thì hàm $f$ không thể là hàm tuần hoàn khi $n$ càng lớn.

Nếu $|k|\leq 2$ đặt $k=2\cos(x)$ thì $u_n=\frac{\sin(nx)}{\sin(x)}$ ( hỏi mấy thánh về dãy )

$\Rightarrow f(n)=\frac{\sin(nx)}{\sin(x)}f(1)-\frac{\sin((n-1)x)}{\sin(x)}f(0)$

Giả sử hàm $f$ tuần hoàn theo chu kì $p$ thì ta có $f(n)=f(n+p)$

$\Rightarrow \frac{\sin(nx)}{\sin(x)}f(1)-\frac{\sin((n-1)x)}{\sin(x)}f(0)=\frac{\sin(nx+px)}{\sin(x)}f(1)-\frac{\sin((n-1)x+px)}{\sin(x)}f(0)$

Giờ ta đồng nhất thức ( không biết có được xài không :v ) thì sẽ ra kết quả là $px=2q\pi$

$\Rightarrow k=2\cos(t\pi),t=\frac{2q}{p}$ :D




#544520 CMR trong dãy số $m,f(m),f\left ( f(m) \right ),...$ co...

Gửi bởi Idie9xx trong 16-02-2015 - 19:03

xét hàm số $f(n)=\left [ n+\sqrt{n} \right ] \ \ \forall n\in \mathbb{N}^*$.Cho $m\geq 1$ là số tự nhiên.Xét dãy các số $m,f(m),f\left ( f(m) \right ),...$

CMR trong dãy số có vô hạn số chính phương 

 

U-Th

1. Xét các số nguyên dương $n$ có dạng $t^2+a$ với $t\geq a>0$ ta có

$f(t^2+a)=[t^2+a+\sqrt{t^2+a}]= t^2+t+a$

$f(t^2+t+a)=[t^2+t+a+\sqrt{t^2+t+a}]=t^2+2t+a=(t+1)^2+(a-1)$

Từ đó ta dễ dàng chứng minh được $f^{(2a)}(t^2+a)=(t+a)^2$ ($f^{(k)}(n)=f(f(f(...(n)...)))$ có $k$ lần $(..)$)

2. Xét các số nguyên dương $n$ có dạng $t^2-a$ với $t>a\geq 0$ ta có

$f(t^2-a)=[t^2+a+\sqrt{t^2-a}]=t^2+t-a-1$

Mà $t^2+t-a-1$ thuộc dạng 1 rồi nên ta chứng minh được $f^{(2(t-a)-1)}(t^2-a)=(2t-a-1)^2$

Từ các trường hợp 1 và 2 ta rút ra nhận xét với mỗi số nguyên dương $m$ ta luôn tìm được các số nguyên dương $k,p$ thỏa $f^{(k)}(m)=p^2$ từ đó chứng minh được yêu cầu của đề bài :)




#544503 CMR $f(n )=g(n)\ \ \forall n\in \mathbb{N...

Gửi bởi Idie9xx trong 16-02-2015 - 17:59

Cho $f:\mathbb{N}^*\rightarrow \mathbb{N}^*$ là toàn ánh và $g:\mathbb{N}^*\rightarrow \mathbb{N}^*$ đơn ánh thỏa mãn $f(n)\geq g(n)\ \ \forall n\in \mathbb{N}^*$

CMR $f(n )=g(n)\ \ \forall n\in \mathbb{N}^*$

 

U-Th

Giả sử tồn tại số $m_0$ thỏa $f(m_0)>g(m_0)$

Do $f$ toàn ánh nên tồn tại một số $m_1$ thỏa $f(m_1)=g(m_0)$

Mà $f(m_1)\geq g(m_1)\Rightarrow g(m_0)>g(m_1)$ ( do $g$ đơn ánh)

Dễ thấy sẽ có một dãy vô hạn $(m_t)$ thỏa $g(m_0)>g(m_1)>...>g(m_{t-1})>g(m_t)>...$

Điều này là vô lí bởi dãy $g(m_t)$ bị chặn. Nên ta có $f(n)=g(n),\forall n \in \mathbb{N}^*$ đpcm




#535214 $f(x-f(y))=2f(x)+x+f(y)$

Gửi bởi Idie9xx trong 28-11-2014 - 19:56

Tìm hàm số $f: \mathbb{R} \to \mathbb{R}$ thỏa mãn

$f(x-f(y))=2f(x)+x+f(y)$

Cho $x=f(y)$ ta có $f(0)=2(f(f(x))+f(x))$ thay $x=0$ có $2f(f(0))=-f(0)$

Thay $x,y$ lần lượt bằng $x+f(x),x$ ta có $2f(f(x)+x)=-(f(x)+x)$

Từ hai điều trên thay $x$ bằng $f(x)\Rightarrow f(\frac{f(0)}{2})=-\frac{f(0)}{4}$

Cho $x=0,y=f(0)$ có $f(-f(f(0)))=2f(0)+f(f(0))\Rightarrow f(\frac{f(0)}{2})=2f(0)-\frac{f(0)}{2}$

$\Rightarrow -\frac{f(0)}{4}=2f(0)-\frac{f(0)}{2}\Rightarrow f(0)=0$

Cho $y=0\Rightarrow f(x-f(0))=2f(x)+x+f(0)\Rightarrow f(x)=-x$

Vậy hàm thỏa mãn đề bài là $f(x)=-x$ :))




#534327 $f(2x-f(x))=x$

Gửi bởi Idie9xx trong 23-11-2014 - 10:20

Vậy mình có thể hỏi là lời giải cuối cùng của bài này là như thế nào vậy ạ. :)

Đã chỉnh sửa ở trên. Bạn có thắc mắc gì nữa không :)




#534305 $f(2x-f(x))=x$

Gửi bởi Idie9xx trong 23-11-2014 - 07:37

Chỗ $a+nc=a'+m'c$ , vì $a,c,a',c'$ là số thực thì sao lại luôn có thể tìm được các số nguyên $m,n$ thỏa mãn đẳng thức đó vậy ? mà câu ( do hàm liên tục có thể chỉ sấp sỉ nhau nghĩa là gì vậy ?)

 

 

cái kĩ thuật mà tạo ra được $f(b)-b=c$ là gì vậy, và khi nào thì áp dụng được nó vậy @@

Do lỗi kĩ thuật rồi :v em nhờ hồi trước có dùng một cái tương tự này nên áp dụng xem thế nào ai ngờ sai ~.~ thực ra nên dùng cái chứng minh $f$ là hàm tăng kia :))




#534304 CMR không tồn tại hàm $f$ và $g$ thỏa $f\l...

Gửi bởi Idie9xx trong 23-11-2014 - 07:19

Có phải là cho $x,y$ phân biệt và bé xíu thì phần nguyên của nó đều bằng 0 phải không, và khi cộng vào t thì không làm thay đổi phần nguyên của t ?

Bé xíu như $0,02$ thì phần nguyên bằng $0$, khi cộng $t$ vào thì cũng tùy theo $t$ mà 2 phần nguyên của nó có bằng nhau hay hơn kém nhau 1 đơn vị :)




#534275 $f(2x-f(x))=x$

Gửi bởi Idie9xx trong 22-11-2014 - 21:40

Tìm hàm liên tục $f: R\rightarrow R$  thoả mãn:
  1) $f$ là đơn ánh

  2) $f(2x-f(x))=x$

  3) Tồn tại $x_{0}$ thỏa mãn $f(x_{0})=x_{0}$

Từ $2)$ ta thấy $f$ toàn ánh, kết hợp với đơn ánh và liên tục nên $f$ đơn điệu.

Giả sử hàm là hàm giảm ta có: $x>y \Rightarrow f(2x-f(x))>f(2y-f(y))$

$\Rightarrow 2x-f(x)<2y-f(y) \Rightarrow f(x)-f(y)>2(x-y)>0 \Rightarrow f(x)>f(y)$ vô lí.

Nên $f$ là hàm tăng. $(*)$

Ta có: $f(a+(a-f(a)))=a$ Giả sử $a-f(a)=c\neq 0$ thì $f(a+c)=a$

Với $b=a+c$ thì $b-f(b)=c$

Vậy ta có $f(a+nc)=a+(n-1)c$

Nếu $a>x_0$ mà $c<0$ thì tồn tại $n$ thỏa $a+nc<x_0$ và $a+(n-1)c>x_0$

Khi đó $f(a+nc)<f(x_0)=x_0<a+(n-1)c$ mâu thuẫn.

$\Rightarrow a>x_0$ thì $c> 0$

Giả sử ta có $a'$ thỏa $a'-f(a')=c'\neq 0,c'< c$ thì tồn tại 2 số $m,n$ nguyên dương thỏa:

$a+nc>a'+mc'$ mà $a+(n-1)c<a'+(m-1)c' \Rightarrow f(a+nc)<f(a'+mc')$ không thỏa mãn $(*)$

Chứng minh tương tự nếu có $c'>c$

Nên $x-f(x)=c, \forall x>x_0$ dễ thấy tồn tại $x$ thỏa $x-c<x_0$ không thỏa mãn $(*)$

$\Rightarrow f(x)=x, \forall x>x_0$

Chứng minh tương tự với $x<x_0$ ta có được $f(x)=x, \forall x<x_0$

Kết luận hàm thỏa là $f(x)=x$ 

Ps: $a+nc>a'+mc'$ mà $a+(n-1)c<a'+(m-1)c'$ có nghĩa là $c-c'>(a+nc)-(a'+mc')>0$

Không biết có còn sai chỗ nào không :wacko:




#534231 Tìm hàm liên tục $f(f(x)+y)=2x+f(y)$

Gửi bởi Idie9xx trong 22-11-2014 - 19:23

Mình thấy pt cauchy có dạng f(x)+f(y)=f(x+y) đúng với mọi x,y thuộc R, nhưng ở đây m,n là thuộc tập giá trị của ánh xạ f, chưa chạy toàn R thì cũng được áp dụng à, bạn giải thích rõ với, mình không hiểu lắm.

Do $f(f(x))=2x$ nên $f$ toàn ánh. Bài giải trên thiếu cái này.




#533829 CMR không tồn tại hàm $f$ và $g$ thỏa $f\l...

Gửi bởi Idie9xx trong 19-11-2014 - 21:51

chứng minh rằng không tồn tại cặp hàm số $f:\mathbb{R}\rightarrow \mathbb{R}$ và $g:\mathbb{R}\rightarrow \mathbb{R}$ thỏa mãn đồng thời các điều kiện sau

$i)$  $g$ tuần hoàn

$ii)$ $f\left ( \left \lfloor x \right \rfloor \right )+g(x)=x^3$

 

NTP

Giả sử hàm $g(x)$ tuần hoàn chu kỳ $t$ ta có:

$f\left ( \left \lfloor x+t \right \rfloor \right )-f\left ( \left \lfloor x \right \rfloor \right )=(x+t)^3-x^3=t(x^2+xt+t^2)$

Ta thấy tồn tại 2 số $x,y$ phân biệt thỏa $\left \lfloor x \right \rfloor=\left \lfloor y \right \rfloor,\left \lfloor x+t \right \rfloor=\left \lfloor y+t \right \rfloor,x+y+t \neq 0$

Ta có $f\left ( \left \lfloor x+t \right \rfloor \right )-f\left ( \left \lfloor x \right \rfloor \right )=t(x^2+xt+t^2)$

và $f\left ( \left \lfloor y+t \right \rfloor \right )-f\left ( \left \lfloor y \right \rfloor \right )=t(y^2+yt+t^2)$

Do $\left \lfloor x \right \rfloor=\left \lfloor y \right \rfloor,\left \lfloor x+t \right \rfloor=\left \lfloor y+t \right \rfloor$

nên $\Rightarrow t(x^2+xt+t^2)=t(y^2+yt+t^2) \Rightarrow t(x-y)(x+y+t)=0 \Rightarrow t=0$ vô lí

Nên không tồn tại cặp hàm $f,g$ thỏa mãn (dpcm) >:)




#533814 Biết $ \sum_{1}^{\infty }\frac{1...

Gửi bởi Idie9xx trong 19-11-2014 - 21:06

$ \sum_{1}^{\infty }\frac{1}{ n^2 } =\frac{ \pi^2 }{6}$ Tính $ \sum_{1}^{\infty }\frac{ 1 }{ (2n-1)^2 } $

$S=\sum_{1}^{\infty }\frac{1}{ n^2 } =\frac{ \pi^2 }{6}$

$T=\sum_{1}^{\infty }\frac{ 1 }{ (2n-1)^2 }$

Ta có $\dfrac{S}{4}=\sum_{1}^{\infty }\frac{ 1 }{ (2n)^2 }$

$\Rightarrow T+\dfrac{S}{4}=\sum_{1}^{\infty }\frac{ 1 }{ (2n-1)^2 }+\sum_{1}^{\infty }\frac{ 1 }{ (2n)^2 }=\sum_{1}^{\infty }\frac{1}{ n^2 }=S$

$\Rightarrow T=\dfrac{3}{4}S=\dfrac{\pi^2}{8}$ :lol:




#484444 $f(x^2+f(y))=y+f^2(x),\;\;\forall x,y\in \mathb...

Gửi bởi Idie9xx trong 23-02-2014 - 20:12

Còn cái bài hàm quen thuộc mà anh nói, anh có thể làm hoặc hướng dẫn em được không ạ ?

$f(x^2)=(f(x))^2\geq 0$ nên với $x> 0 \Rightarrow f(x)\geq 0$

Cho $x>y$ ta có

$f(x)=f(y)+f(x-y)\geq f(y)$, vậy nên $f$ là hàm tăng.

$f(x+y)=f(x)+f(y)\Rightarrow f(x)=cx,\forall x\in \mathbb{Q}$ thử lại thấy $c=1$ thoả.

Với mỗi số vô tỉ $r$ luôn tồn tại hai dãy số hữu tỉ $(p_n),(q_n)$ thoả

$\lim_{n\rightarrow +\infty} q_n=\lim_{n\rightarrow +\infty} p_n=r$ và $q_n>r>p_n,\forall n \in \mathbb{N}$

Ta có $f(q_n)\geq f(r)\geq f(p_n)$ khi $n\rightarrow +\infty \Rightarrow f(r)=r$

$\Rightarrow f(x)=x,\forall x\in \mathbb{R}$ :))




#484387 $f(x^2+f(y))=y+f^2(x),\;\;\forall x,y\in \mathb...

Gửi bởi Idie9xx trong 23-02-2014 - 16:33

Tìm tất cả các hàm số $f:\mathbb{R}\rightarrow \mathbb{R}$ và thỏa mãn :

$$f(x^2+f(y))=y+f^2(x),\;\;\forall x,y\in \mathbb{R}\;\;\;\;(1)$$

 

Mình chứng minh được $\left\{\begin{matrix} f(x^2)=f^2(x),\;\forall x\in \mathbb{R}\\ f(x+y)=f(x)+f(y),\;\;x,y\in \mathbb{R} \end{matrix}\right.$

Sau đó tính biểu thức $f((x+1)^2)$ theo hai cách thì nó ra thế này $2f(x)(1-f(1))=f^2(1)$ ????

Rõ ràng hàm $f(x)=x$ thỏa $(1)$. Không biết sai sai chỗ nào @@.

....

Ta sẽ tính biểu thức $f((x+1)^2)$ theo $(2)$ và $(3)$ :

$f((x+1)^2)=f^2(x+1)=(f(x)+f(1))^2=f^2(x)+2f(x)f(1)+f^2(1),\;\forall x\in \mathbb{R}$

$f((x+1)^2)=f(x^2+2x+1)=f(x^2)+2f(x)+f(1)=f^2(x)+2f(x)+f(1),\;\forall x\in \mathbb{R}$

Từ hai kết quả này ta suy ra :

$f^2(x)+2f(x)f(1)+f^2(1)=f^2(x)+2f(x)+f(1),\;\forall x\in \mathbb{R}\Rightarrow 2f(x)(1-f(1))=f^2(1)\;\;(*)$ [/hide] 

 

Ai có cách nào hay thì làm luôn nhé ! 

Từ $(2)$ và $(3)$ có thể suy ra luôn là $f(x)=x$ rồi mà :P ( một bài hàm quen thuộc :closedeyes:  )

Còn cái $(*)$ là do viết thiếu, phải là $f^2(1)-f(1)$ :lol:  bạn trừ lại thử xem :)




#483393 $$f(f(f(n)))= f(n+1)+1$$

Gửi bởi Idie9xx trong 16-02-2014 - 09:21

Bài toán :

Tìm hàm số $f \, :\, \mathbb{N}\to \mathbb{N}$ sao cho : $$f(f(f(n)))= f(n+1)+1\,\,\forall n\in\mathbb{N}$$

Dễ thấy với mỗi số tự nhiên $k$ luôn tồn tại 1 số tự nhiên $m$ thỏa $f(m)=f(k)+1$ :ohmy:

Bằng qui nạp ta chứng minh được rằng với mỗi số tự nhiên $t>f(k)$ thì tồn tại số tự nhiên $u$ sao cho $f(u)=t$ :lol:

Giả sử ta có số $a$ sao cho $f(a)=0$ thì tồn tại số tự nhiên $b$ sao cho $f(f(b))=a\Rightarrow f(f(f(b)))=0\Rightarrow f(b+1)=-1$ (mâu thuẫn)

Nên không tồn tại $a$ thỏa $f(a)=0$

Giả sử ta có số $a$ sao cho $f(a)=1$ thì tồn tại số tự nhiên $b$ sao cho $f(f(b))=a\Rightarrow f(f(f(b)))=1\Rightarrow f(b+1)=0$ (mâu thuẫn)

Nên không tồn tại $a$ thỏa $f(a)=1$

Chứng minh qui nạp với điều giả sử tương tự ta thấy không có hàm thỏa đề :mellow:

Vậy không có hàm thỏa mãn đề bài :ukliam2:  :))

-----------------

WS : Với hàm $f(n)=n+1$ thì sao a ?